Math 153 diary, fall 2009: third section
Earlier material
Much earlier material
In reverse order: the most recent material is first.


Thursday, December 10 (Lecture #28)

Area between two curves
Here's the final topic of the semester, which is a simple introduction to the uses of the definite integral. The definite integral has hundreds of important applications in science and engineering.

Suppose we are given two functions defined on an interval, Top(x) and Bot(x) ("Bot" is an abbreviation for "Bottom") and we know on that interval that Top(x)>Bot(x). How can we compute the area enclosed by the two curves and by vertical lines on the sides of the interval?

I always imagine that the interval is chopped into lots of little pieces, each of length dx. Then these pieces chop up the area, as shown in the picture to the right. Each little slice of area is almost a rectangle (if we ignore the possible tilts at the top and bottom). The area of the approximate rectangle is [Top(x)–Bot(x)] (the length of the vertical side, the difference in the heights of the graphs) multiplied by dx, the width. Now I need to take the Sum of these approximating slices, and add them up from a to b, or, as I think, from Left to Right. Therefore I believe that the area between these curves on this interval is
LeftRight[Top(x)–Bot(x)]dx.

Random example #1
This example is not random, but it is constructed so I could do it easily in class. I would like to find the area of the region enclosed by the line y=x+2 and the parabola y=x2. I would probably try to begin almost any problem of this type by sketching the region first. The picture to the left is such a sketch. Where do these curves intersect? Well, we need to find x's which give the same y, so we solve x2=x+2 so that x2–x–2=0 and this is (wow! Oh, hold on, he made it so it would work out) x2–x–2=0 so that (x–2)(x+1)=0. The roots are x=2 and x=–1.
I honestly think that the phrase "the region enclosed by the line y=x+2 and the parabola y=x2" specifies exactly one piece of the plane. You can argue with me -- I discuss this a bit more in the example below. Here I think we have Left=–1 and Right=2 and Top(x)=x+2 and Bot(x)=x2. So let's compute:
LeftRight[Top(x)–Bot(x)]dx=∫–12[{x+2}–{x2}]dx=
(1/2)x2+2x–(1/3)x3|–12=[(22/2)+2·2–(8/3)]–[((–1)2/2)+2·(–1)–(–8/3)].
This turns out to be 9/2. I hope. And, by the way, if you look at the picture, I hope you can see that the region fits inside a box with height 4 and width 3, so the area should be less than 3·4=12, which it is. Getting some approximate idea of the answer is useful, if you, like me, make sign errors and ... well, other kinds of errors.

Random example #2
Again, this is an arranged example and everything will work out neatly. Real problems are rarely like this. I'd like to compute the area of the part of the plane which is between the parabolas y=x2–5 and y=3–x2. The first curve opens up and has its bottom at (5,0). The second opens down, with its top at (3,0). A picture of the two curves is shown to the right.

I mentioned in class that it is certainly possible to "misunderstand" the statement of this problem, if you work at it. The plane is actually divided into five different regions by these two curves. And maybe someone could maybe declare that there's more than one candidate for the region between the parabolas (maybe all five are candidates, or maybe only three of them are?). I guess maybe I might agree if you really argue with me about it, but if the discussion is just a way to get out of computing the area of the only region with finite area then I would not agree. Please compute the area of that region, and then, later, argue about it.

The curves intersect where x2–5=3–x2. Since this is a problem in a math class, let's see: that's 2x2=8 or x2=4 so that x=+/–2. I think that Left=–2 and Right=+2.

Something new!?
There is a little bit that's new here. Notice that if Top(x)=3–x2 and Bot(x)=x2–5, then actually Bot(x) is always negative in the interval [–2,2]. Even so, the quantity Top(x)–Bot(x) gives the geometric length of the vertical side of a (dx) thin slice of area. Top(x)–Bot(x) will be positive. In fact, if you look really carefully at the picture, you'll see that there are two pieces of [–2,2] where both Top(x) and Bot(x) are negative. But Top(x) is still bigger than Bot(x), so Top(x)–Bot(x) will be positive, even though both of them are negative. The geometric length is still Top(x)–Bot(x).

The computation
LeftRight[Top(x)–Bot(x)]dx=∫–22[{3–x2}–{x2–5}]dx=
–22[{8–2x2]dx=8x–(2/3)x3|–22=[(8·2)–(2/3)23]–[(8·–2)–(2/3)(–2)3]=64/3.
This is less than a box which is 8 units high and 4 units wide (such a box would enclose the entire area). Also, several students remarked that we could have computed the integral from 0 to 2 and doubled the result, to take advantage of symmetry.

Random example #3
I would like to find the geometric area between the sine and cosine curves over one period of these functions. So I drew a picture similar to what's shown to the right. Again, it may be possible to misunderstand the question since this situation is more complicated than previous ones. But "one period" is 2Π here, and the word "between" in this situation refers, in fact, to three specific regions in the picture.

So the area which I wanted to compute is shown to the right. Here is what I thought that I needed to do (clear evidence that the brain was not totally functional!). I thought that I would need to split the area computation into three pieces, and compute three different integrals, and then add them up. The idea would be as shown to the left where first one formula then the other, and then the other again, is on top. Well, this could be done but it would actually be much more work than is needed.

A student (who?) kindly pointed out that the left-most region and the right-most region when put together are congruent to the central region: so the sum of the areas of these two regions must be equal to the area of the region in the middle. Thank you!

The curves intersect where sin(x)=cos(x). This is at x=Π/4 and x=5Π/4. So Left=Π/4 and Right=Π/4 and between these numbers, the sine curve will be Top(x) and the cosine curve will be Bot(x). So I need to compute twice the definite integral over this interval to get the total area requested. Here we go (and be careful with minus signs!):
    2∫Π/45Π/4(sin(x)–cos(x))dx=–cos(x)–sin(x)|Π/45Π/4=[–cos(5Π/4)–sin(5Π/4)]–[–cos(Π/4)–sin(Π/4)]=
    [–{–sqrt(2)/2}–{–sqrt(2)/2}]–[–{sqrt(2)/2}–{sqrt{2}/2}]=4sqrt(2)
There were only SEVEN minus signs in the next-to-last expression. Mistakes are easy!

Another ...
How much geometric area is enclosed by y=x(x–1)(x–3) and the x-axis? Again, a graph is shown to the right, and these bumps definitely have different sizes. But my patience (?) is running out. In a spare tenth of a second, we get:


> int(x*(x-1)*(x-3),x=0..1);
            5/12

> int(x*(x-1)*(x-3),x=1..3);
            -8/3
So the geometric area is 5/12+8/3, which is, I can tell you, 37/12. So this is ∫03|x(x–1)(x–3)|dx (the absolute value signs make both bumps "sit" on top of the x-axis. I did compute these integrals in class, and the first step was multiplying out x(x-–1)(x–3) so I could get the antiderviative easily.

Sideways?
I asked what does the curve defined by the equation (y–1)(y–2)=x look like? With a bit of thought, we decided that it was a parabola with axis of symmetry parallel to the x-axis, and that it was opening to the right. A graph is shown to the right.

Notice, please, that the point (2,0) is on the graph, as well as the point (0,2). Therefore there's a (exactly one!) straight line through these two points. Its equation is x+y=2 (Admission: I sort of guessed at the equation, since the sum of each pair of coordinates is 2, that must be the equation!). What's the area of the region bounded by the line and the parabola?

The dy method
If we have a region in the plane bounded by two curves, x=Left(y) and x=Right(y) (where Left(y)<Right(y) for the y's of interest here) and also bounded by lines y=Bottom and y=Top, then we could imagine slicing this area by lots of horizontal lines dy apart. The region would be divided into lots of pieces, and each piece would approximately be retangular. The height of the (almost) rectangle would be dy, and the width would be Right(y)–Left(y). So the area would be (Right(y)–Left(y))dy. Since this is the area of a slice, we would get the total area by taking the Sum of the areas of the slices, from the Bottom to the Top: ∫y=Bottomy=Top(Right(y)–Left(y))dy.

In our case ...
Top is 2 and Bottom is 0. Left(y) is (y–1)(y–2)=y2–3y+2 (you should multiply/distribute/expand/foil whatever! before integrating) and Right(y) is 2–y. Thus the area can be computed by
y=Bottomy=Top(Right(y)–Left(y))dy=
y=0y=2([2–y]–[y2–3y+2])dy= ∫y=0y=2([2y–y2])dy=y2–(1/3)y3|02=4–8/3=4/3.

Comments
Certainly this is not the only method of getting this area. You could possibly imagine dissecting the area dx, and chopping it up somehow so the computation could be done. You would need to solve for x in terms of y, which is possible in this case but maybe not too easy if the equation were more complicated. Also that would be much more work, I think. Or maybe you just interchange x and y, and then redraw the graph, and do it dx. That's certainly possible. I just report, though, that for many students in the class (physics students, engineering students) many other geometric and physical quantities will occur (moment of inertia, center of gravity, etc.) which can be computed more naturally dy, and therefore "chopping up" regions in this way will be useful.

Winding up ...
Please study for the final exam.


Tuesday, December 8 (Lecture #27)
Some derivatives
I began by asking what the derivative of 17x was. There was some confusion, so we did this:
    1. y=17x
    2. (Take logs or lns) ln(y)=ln(17x)
    3. ln(y)=x·ln(17)
    4. ln(y)=x·ln(17)
    5. (d/dx the equation) (1/y)(dy/dx)=ln(17)
    6. (Solve for dy/dx) dy/dx=ln(17)y
    7. dy/dx=ln(17)1717
This is worth going through if only for the psychology of differentiating x·ln(17) (the right-nad side of the transition from 4 to 5 above). Apparently it is sometimes difficult for people to realize that ln(17) is a constant. Sigh.

The derivative of 17x is (ln(17))17x. And the derivative of 556x is (ln(556))556x. The derivative of x400400x is 400x399400x+x400(ln(400))400x. The derivative of 37sec(5x2) is (ln(37))37sec(5x2)sec(5x2)tan(5x2)10x.

All this was preparation for understanding the solutions to a differential equation and for manipulating these solutions, using ln and exp.

What are the solutions?
I would hope that dy/dt=ky would have a family of solutions (the general solution) and we would use an initial condition to pick out one of these (the particular solution).
Well, first let's guess one solution of dy/dt=ky. For example, I know a wonderful function which is its own derivative: the exponential function (so that's a solution when k=1). After some further consideration, we could try the function ekt. Indeed, if we differentiate this, the Chain Rule "spits out" a multiplicative factor of k, so the derivative of ekt is ektk, and this is ky.

Are there other solutions? Here is a trick to learn about other possible solutions. It is a fairly clever trick, and is used in other computations, which is why I'm showing it to you. Suppose y is another solution of dy/dt=ky. I want to compare it to the solution we know, ekt. The trick is to compare this way:
Look at y/ekt (the unknown solution y divided by the known solution, ekt). Let's differentiate this. The Quotient Rule gives us:
    y´(ekt)–y(ektk)
    -------------------
           (ekt)2
Look carefully at the top of the fraction. We are assuming that y´ is ky. Then the top becomes:
y´(ekt)–y(ektk)=ky(ekt)–y(ektk)=0
because things exactly cancel. This means (MVT tells us: a function with 0 derivative is constant) y/ekt is a constant, C. So y=Cekt.

Cekt
I admit totally truthfully that I don't go through any process remotely like what I just showed you in practice. In fact, if I think that the differential equation dy/dt=kt is a good description of a situation, then I immediately jump to Cekt. If you work with such situations for a while, I think you will do this also.

Student-suggested bacteria
We got some bacteria with the following (hypothetical) observations of a bacterial colony (I forget the actual numbers we used in class so I will just invent some here):

Initially, there are 50 bacteria. In 3 hours, 100 bacteria are observed (3 hours is called the doubling time). In 6 hours, 200 bacteria are observed.
People believe that dy/dt=kt is a differential equation which appropriately describes growth in such a situation (some comments on this assumption are below!) because bacteria (generally) reproduce asexually by division, and new bacteria are created by a fraction of the current bacteria splitting. I would like to make a mathematical model (in this case, get a simple formula) for the bacteria at time t.

I will measure t in hours from the start of the observation. B(t) will be the number of bacteria at time t. So B(0)=50 and B(3)=100 and B(6)=200. If we suppose that B(t)=Cekt (that follows from the differential equation's applicability to this situation) then we need to identify numbers for C and k.

Since B(0)=50 and B(0)=Cek·0=Ce0=C·1=C. So C is 50. Now we know that B(t)=50kt. We need to identify k, and we can use B(3)=100 for this. So: 100=50ek(3) which is 2=e3k which is (take ln's!) ln(2)=3k so that k=[ln(2)/3]. (People who work with these equations a great deal develop lots of computational shortcuts!)

So the number of bacteria is given by B(t)=50e[ln(2)/3]t. We could check this formula by plugging in t=6, since we know the answer should be 200. Here:
B(6)=50e[ln(2)/3]6=50e2ln(2)=50eln(4)=50·4=200 (because exp and ln are inverse functions).

Defects of the model?
Maybe bacteria don't reproduce the way we think (indeed, when I was younger, mostly people did believe bacteria could only reproduce asexually, but this is not always true). There's a more subtle assumption hidden in the model. The exponential function grows very fast. For example, there will be 1010 bacteria in about 82 hours (I did these computations secretly). That isn't so big, biologically, since there may be about 1014 cells in the human body. But if you continue to believe this model is valid, there will be 10100 bacteria after about five and a half weeks. Now to understand 10100 is difficult. Uh ... there are about, say, 6 billion people in the world, and so there are about 6·109·1014=6·1023 human cells in the world. Multiplied exponentials add "upstairs". This means we'd need lots and lots of worlds (more than 1075) to have maybe one bacteria per cell.

In fact, the bacteria will grow exponentially as long as conditions allow. That is, there needs to be adequate food, places to excrete poisons, etc. I think, in reality, these limits to growth impose themselves in a realistic way fairly soon in the case of bacterial growth. So the exponential model for bacterial growth is valid, but really only for relatively brief intervals. Exponential decay (with k<0) can be applied more appropriately over long periods of time to radioactivity. I'll try to discuss this next time.

Exponential growth: a textbook problem
This is problem #16 of section 5.8: An insect population triples in size after 5 months. Assuming exponential growth, when will it quadruple in size?
I added the following question:
    What is the doubling time of this type of insect?

Solution
The phrase "Assuming exponential growth" means that, if B(t) gives the number of bugs at time t, then we should assume that B(t)=Cekt. So (units!) B(t) will represent the population of the insects at time t measured in months since the start. We know that B(0)=Ck·0=C and B(5)=3C. So 3C=Cek·5, and if we divide by C and ln both sides and divide by 5, we get k=ln(3)/5.
So B(t)=Ce[ln(3)/5]t and we want to know when the population quadruples, that is, reaches 4C. (By the way, it is a good idea to have some estimate in mind just so we can check the answer, or, if the answer is correct and way off the estimate, we can improve estimating skills. In this case, I would guess that about another month or so is needed to get to four times the initial population: so my guess is 6, maybe a little bit more.) So let's solve 4C=Ce[ln(3)/5]t: we divide by C, ln both sides, and divide by [ln(3)/5]. The result is t=[5ln(4)/ln(3)]≈6.31 months.

I had also asked "What is the doubling time?" Doubling time is a standard measure of growth, and is the time needed for the population to double (indeed!). If the formula governing growth rate is exponential, then doubling time is a constant (hey, if the population was given by t2+1, then the population doubles from 0 to 1, and it also doubles from 1 to sqrt(3): the time intervals change!). In this case, we are told that population is given by an exponential formula and that a population quadruples in 6.31 months. I think the doubling time is 3.15 months.

Exponential decay: a textbook problem
This is problem #17 of section 5.8: A 10-kg quantity of a radioactive isotope decays to 3 kg after 17 years. Find the decay constant of the isotope.
I added the following questions:
    Also, what's the half-life? What is half-life?

Solution
Well, suppose R(t) is the quantity in kg of the radioactive substance at time t in years. We assume (and mostly this is true) that amount of the substance is given by R(t)=Cekt. We know that R(0)=10, so that C=10 (always try to start the "clock" in these problems at 0 so that C will be the initial amount). Also since R(17)=3, 10ek·17=3. Now divide by 10, ln both sides and divide by 17: k=ln(3/10)/17. I think this is the decay constant. Please notice that since 3/10 is less than 1, ln(3/10) is negative so the constant in the exponential formula is negative, and this is, indeed, decay.

The half-life of a radioactive substance is the time needed for an initial quantity to reduce to half. Since we are told that in 17 years, 10 kg reduces to 3 kg, I am sure that a half-life of this substance will be less than 17. In fact, a half-life will be more than, say, 8, since a half-life of 8 would result in 2.5 kg at 16. So a casual estimate of half-life is somewhere between 8 and 17.
Here we want t so that 10e[ln(3/10)/17]t=5. Divide by 10, take lns, and divide by [ln(3/10)/17]: the result is t=ln(1/2)/[ln(3/10)/17]. This is about 9.79.

Radioactivity: a very short discussion
There are lots of radioactive isotopes. A web page I found mentions these "widely used industrial isotopes ... 192Iridium with a half-life of 74 days ... and 60Cobalt with a half-life of 5.3 years". If radioactive substances have the same activities (there are three principal types: alpha and beta particle emissions and gamma rays) then it is likely that a substance with a shorter half-life will be more dangerous than one with a longer half-life. But things can get more complicated and half-life is not the only consideration. Please don't think that I know more than a superficial amount about radioactivity. There's also tritium, which is an isotope of hydrogen, 3H, which was widely used to make paint glow (signs, rifle sights, sometimes as a tracer in the body). It has a half-life of 12.3 years, although one source I found declared that when used as part of a biological sample (administered as water) then the "biological half-life" (a phrase I had not seen before) in humans was about 10 days (this refers not to the decay of the tritium, which takes quite a while, but to how a normal human body processes water). Tritium is not good in the body.

14C
Radiocarbon dating (which I tried to explain and was not too successful) is one of the most widely used methods of dating human remains and cultural objects. Please follow the link supplied to read about it a bit. The basic assumption is that an organic (living) object has a certain percentage of 14C ("Carbon 14") and this percent is not replenished which the object is dead. So as the 14C decreases, a reasonable guess about the age of the object can be made. 14C is convenient because there's a good amount of carbon in most organic objects, and because the 14C half-life is commensurate (appropriate amount) to measure much of human history.

One source I found said that the half-life of 14C is about 5,730 years. The text asserts that the decay constant for 14C is about –.000121. Are these two numbers consistent? Is there some way to check? Well, suppose 14C follows a Cekt formula. If C=1 and k=–.000121 and t=5,730, then Cekt should be about a half. In fact, we computed this (or rather, calculators did) and the result is .499905, which is quite close to a half. I was happy.

The final exam
I will discuss what I regard as a serious difficulty of many 153 students: knowing how to study. I certainly don't want to deny talent and intelligence as factors, but knowling how to study is important. I've probably had extended discussions with about half the students in this class and have been stunned by widely held attitudes which I believe actively block success (things like, "I don't open the book ... I can't work with other students ..."). I've tried to make suggestions and the suggestions mostly have been ignored. I remark that, well, your way doesn't seem to work too well, and maybe a sane person might want to try another way. My suggestions aren't exciting (indeed, I used the word "boring" in class) but I really believe that, with high probability, they lead to success.

I read and commented on the contents of .the web page I wrote supporting your study for the final exam.


Thursday, December 3 (Lecture #26)
Movement of a particle
Let's assume that the velocity of a particle traveling on the x-axis is given as a function of time: v(t)=4t2–t4 for t between 0 and 5. (As far as I know, this is a rather non-physical example, but I hope that the questions and ideas contributing to the solutions are correct and useful.)
  1. For which values of t is the particle traveling to the right? To the left?
    "Travel to the right" in terms of velocity means "v(t)>0" and of course "to the left" is "v(t)<0". Since v(t)=4t2–t4=t2(4–t2). Notice that t2 is non-negative, so that 4–t2 has the sign information. We should consider 4–t2 on the time interval [0,5]. Of course, 4–t2=0 when t is +/–2. So (considering everything) v(t)>0 when t is between 0 and 2 and v(t)<0 when t is between 2 and 5. Notice that velocity is 0 when t=0 and t=+/–2.

    Here is where I stopped last time. I want to investigate and contrast the ideas of displacement and distance traveled. This might be interesting to physics people.

  2. At what time is the particle farthest to the right?
    The particle travels to the right when t is between 0 and 2, and travels to the left when t is between 2 and 5. So the particle is farthest to the right when t=2.
  3. Suppose that at time 0 the particle's position is 12. What is its rightmost position?
    Well, displacement in an interval [t1,t2] means the difference in position at time t1 and at time t2. So the displacement is the definite integral of the velocity over the interval [t1,t2]. So the displacement over the interval [0,2] (2 is when the particle is farthest to the right) is ∫02v(t) dt=∫024t2–t4dt=(4/3)t3–(1/5)t5|02=((4/3)(23)–(1/5)25)(0 [all 0 when t=0])=(64/15). We get the position when t=2 by adding the initial position, which is 12, to this result. So the position when t=2 is 12+(64/15).
  4. What is the displacement of the particle during the time interval [0,5]?
    The displacement is ∫05v(t) dt: this is the difference in the position at time 5 and the position at time 0. So we compute: it is ∫054t2–t4dt=(4/3)t3–(1/5)t5|05, and this turns out to be (4/3)25–(1/5)55, which is –625+(4/3)25: a quite negative number, so the particle moves really left in the interval.
  5. What is the distance the particle travels during the time interval [0,5]?
    This is, to me, the most ticklish (in the sense of "delicate") question. The particle travels right from 0 to 2 and then left from 2 to 5. The displacement is not the same as the distance traveled. The distance traveled is how the particle goes from 0 to 2 and then (added on) how the particle goes from 2 to 5. The integral from 0 to 2 will be positive and the integral from 2 to 5 will be negative. But we need the sum of the amounts of each, with the signs stripped off. Hey, here are two math ways of writing what we want:
        ∫05|v(t)|dt (notice the absolute value sign!)
    or
        ∫02v(t)dt25v(t)dt.
    The reason for the minus sign is that velocity is negative between 2 and 5. So subtracting the integral of velocity adds the distance traveled then.
    I asked my silicon buddy to compute ∫05|4t2–t4|dt and it did this without much complaint. It used about .17 seconds, which is quite a lot for a simple definite integral. But in that time, it determined the intervals where what's inside the absolute value sign is negative or positive (that's what probably took most of the time), and it chopped the interval into two pieces, switched signs on the formula in one of the intervals, and applied FTC 1 to each of two integrals. The answer was 7003/15.
Here is a sort of picture of the movement of the particle. The picture is qualitatively correct, but the distances are very wrong. I just want you to "see" what the movement might look like.

A "complicated" integral
Let's "compute" ∫01cos(x17)x16dx. If we want to apply FTC 1, we need to find an antiderivative. Well, the integrand is certainly not random. There is a pairing which I hope you notice after more than 90% of a calculus course. The x17 inside the cosine function, and the x16 multiplying the outside of the cosine function. Since we are trying to identify a function whose derivative is cos(x17)x16, to me a natural guess is something like sin(x17). If we make that guess, then the derivative of sin(x17) is cos(x17)(17x16) using the Chain Rule. But we don't want the multiplicative constant 17, so, just as in a bunch of examples we've already done, we fix up our initial guess to get (1/17)sin(x17). And this works, and then, using FTC 1, the definite integral's value is (1/17)sin(x17)|01=(1/17)sin(1).

I'm actually not too interested in the specific value of the integral, but more in how to make the guessing process work. I want to show you how to do this systematically and easily, reducing the chance of error. Of course, the integrand was set up so we could guess. But the process works often enough that people have make some notation which makes it easier to do. Here is what they would write in this case:

In x-land∫cos(x17)x16dx
Guess a uu=x17
Compute dudu=17x16dx
What about the notation?
Well, if u=x17, then du/dx=17x16. This is one of the traditional notations for derivatives ("Leibniz notation"). If we think that du/dx is a fraction, we could multiply by dx and get the equation above. One reason people like Leibniz notation is that it works well with this method of antidifferentiation which is called substitution. Although I know that the derivative is a limit and not a fraction, I certainly use the substitution method very freely, and I don't worry about separating the du and the dx. It works! Notation should help, and this notation works because the Chain Rule is correctly used.
Adjust du to match
what's in the integral
(1/17)du=x17dx
Go to u-land∫cos(x17)x16dx= ∫cos(u)(1/17)du=(1/17)∫cos(u) du
Antidifferentiate(1/17)∫cos(u) du=(1/17)sin(u)+C
Return to x-land(1/17)sin(x17)+C

Of course, this example is constructed so that things work straightforwardly. A chunk of my job is to show you a range of examples and try to help you learn how to recognize situations where this method of antidifferentiation, called the substitution method, will work. Your job, on the other hand, is to practice a bunch of suitable examples (problems in section 5.6).

Another example
Consider ∫x=0x=1x4/(3+x5)dx. I "admit" (?) that this is an invented example, intended to display the substitution method but I also assert that similar computations occur very often in computations coming from "real life". If we want to use FTC, we need to find an antiderivative of x4/(3+x5). This is a slightly complicated function, a rational function (quotient of polynomials). If we take u=3+x5, then du=5x4dx. Well, we have x4dx in the original integral, so (1/5)du=x4dx and the indefinite integral ∫x4/(3+x5)dx in x-land becomes ∫(1/u)(1/5)du in u-land. This is (1/5)∫(1/u)du because constant multipliers can be moved outside the integral sign (a reflection of the derivative of Cf is Cf´). But I can antidifferentiate this. It is (1/5)ln(u)+C. Now back to x-land and get (1/5)ln(3+x5). To finish the computation, remember |01. The result is (1/5)ln(4)–(1/5)ln(3). I would probably leave the answer this way (if I work with it I may make mistakes). Please note that a naive observer when asked for the area under y=x4/(3+x5) and above the x-axis and between y=0 and y=1 would not necessarily expect an expression involving logarithms. The result is, from that point of view, weird, wonderful, unexpected ...

QotD
Find ∫x=0x=sqrt(2)x e(–3x2)dx.

The answer involves ...
6  6  6
Less fearsomely, try u=–3x2. The desired answer can be written as (1/6)–e–6/6.

01sqrt(5x+4)dx
I always try to substitute for the center of the difficulty first (this does not always work, but it helps in lots of examples). In this example I would try u=5x+4. So then du=5dx so (1/5)du=dx, and, as an antiderivative (let me leave out the limits right now):
∫sqrt(5x+4)dx=∫sqrt(u)(1/5)du=∫(1/5)u1/2du=(1/5)(2/3)u3/2+C=(2/15)u3/2+C =(2/15)(5x+4)3/2+C
The outline of this computation is: from x-land to u-land, antidifferentiate, back to x-land. Now if we really want to compute the definite integral, look:
(2/15)(5x+4)3/2|01=(2/15)(93/2)–(2/15)(43/2)
Please notice that the x=0 limit does contribute to this answer -- you can't just ignore it because, golly, x=0. This antiderivative has a non-zero value at x=0.

My silicon pal does a great job on this (it even cleans up the fractions and powers!):

> int(sqrt(5*x+4),x=0..1);
                                      38
                                      --
                                      15

∫x·sqrt(5x+4)dx
As I remarked in class, I think that most people could have made a good guess at the previous antiderivative. I can't do this one without a substitution and some subsequent manipulation. So again let us try the center of the difficulty, u=5x+4, so du=5dx and (1/5)du=dx. But now to get to u-land we need to know how to translate x. Since u=5x+4, we know that u–4=5x and (1/5)(u–4)=x. Therefore:
∫x·sqrt(5x+4)dx=∫(1/5)(u–4)u1/2(1/5)du [to u-land]
(1/25)∫(u–1)u1/2du=(1/25)∫u3/2–u1/2du [pull out constant multiplier, distribute u powers]
(1/25)((2/5)u5/2–(2/3)u3/2)+C=(1/25)((2/5)(5x+4)5/2–(2/3)(5x+4)3/2)+C [antidifferentiate, and back to x-land. Notice that 1/(5/2)=2/5. etc.].
Although it may be possible to check an antiderivative by differentiating, you may not want to!

∫x2sqrt(5x+4)dx
Here I pushed up a power of the x. If you use u=5x+4 again (the center of the difficulty) then as before, (1/5)du=dx and (1/5)(u–4)=x. So the integral becomes:
∫x2sqrt(5x+4)dx=∫[(1/5)(u–4)2sqrt(u)(1/5)du
This is not so pleasant but we can do it. Now [(1/5)(u–4)2=(1/25)(u2–8u+16) and so:
∫[(1/5)(u–4)2sqrt(u)(1/5)du=(1/125)∫u5/2–8u3/2+16u1/2du
So what did I do there? Well, I squared 1/5 to get 1/25, and then multiplied by another 1/5 to get 1/125. Multiplicative constants can be pulled "out" of the indefinite integral. Other kinds of constants can't! Then I distributed the sqrt(u)=u1/2 over the other terms. Powers of u can be antidifferentiated easily, so we get:
(1/125)((2/7)u7/2–(16/5)u5/2+(32/3)u3/2)+C
Now back to x-land:
(1/125)((2/7)(5x+4)7/2–(16/5)(5x+4)5/2+(32/3)(5x+4)3/2)+C
I surely would not want to check this by differentiating! My "pal", in about two-hundredths of a second (.02 seconds) gives this:
> int(x^2*sqrt(5*x+4),x);
                                3/2                     2
                     2 (5 x + 4)    (128 - 240 x + 375 x )
                     -------------------------------------
                                     13125
I hope (I guess!) this is what we have. It sort of looks the same!

∫x·sqrt(5x2+4)dx
Here u=5x2+4 (the most "horrible" thing in the integral), du=10x dx, (1/10)du=x dx (because notice that there is an x inside the integral -- not an accident!). From x-land to u-land, then, we get:
∫x·sqrt(5x2+4)dx=∫sqrt(u)(1/10)du=∫(1/10)u1/2du=(1/10)(2/3)u3/2+C
The x did not just disappear! It was needed in this case to be part of du. And finally we change (1/10)(2/3)u3/2+C to (1/10)(2/3)(5x2+4)3/2+C. You can check this by differentiating, if you use the Chain Rule correctly. The 2/3 cancels the 3/2 coming from the power outside, and the 1/10 cancels the 10 in the 10x coming from what is inside.

∫exsin(5ex+7)dx
Pattern: the worst thing is certainly inside sine. So I will try u=5ex. Then du=5exdx, and (1/5)du=exdx. From x-land to u-land, to antidifferentiate, and back to x-land;
∫exsin(5ex+7)dx=(1/5)∫sin(u)du=(1/5)(–cos(u))+C=(1/5)(–cos(5ex+7))+C.

Three ln integrals
Here are three increasingly irritating antiderivatives involving ln:

  1. ∫[(ln x)2/x]dx
    Solution Take u=ln x (since ln is the center of the difficulty), so du=(1/x)dx, and recognize please that we have dx/x in the integral! So from x-land to u-land:
    ∫[(ln x)2/x]dx=∫(ln x)2[dx/x]=∫u2du
    Now integrate and get u3/3+C, which in x-land is (ln x)3/3+C. Is this good?
  2. ∫[1/{x·ln x)]dx
    Solution Take u=ln x and again du=(1/x)dx. The integral changes:
    ∫[1/{x·ln x)]dx=∫{1/u]du=ln(u)+C=ln(ln(x))+C
    This really is not too horrible. People use functions like this one, really.
  3. ∫[{ln(ln(x))}/{x·ln(x)}]dx
    O.k., this example is more of a textbook example. So, again, I will try u=ln x and again du=(1/x)dx. We will go from x-land to u-land:
    ∫[{ln(ln(x))}/{x·ln(x)}]dx=∫[ln(u)/u]du
    Please notice that the substitution absorbed (?) one of the things in the bottom of the integrand. Now what should we do? If you were "properly conditioned" by the previous examples, you would try a substitution and so I will. What I choose to try (it works, that's why!) is v=ln(u) and dv=[1/u]du. So from u-land to v-land:
    ∫[{ln(u)/u]du=∫v dv=(1/2)v2+C
    Now back and back again:
    (1/2)v2+C=(1/2)(ln(u))2+C=(1/2)(ln(ln(x)))2+C

A differential equation and initial condition again
I'm following the text here. We are next supposed to consider a new sort of differential equation. We have studied such problems as these:
    y´=3x4+{2/x4} and y(1)=2.
This is an initial value problem, with a differential equation and an initial condition. We find what's called the general solution by computing an antiderivative:
∫3x4+{2/x4} dx=∫3x4+2x–4dx=(3/5)x5–(2/3)x–3+C
The perhaps interesting aspects of what was just computed are these: I first changed {2/x3} to 2x–3, which is a more standard way to write powers of x. I regard things like sqrt(x) (just x1/2) and 1/sqrt(x) (surely x–1/2) as invitations to error, directed personally at me. Unfortunately I accept these invitations sometimes. Then I antidifferentiated. Making sure that a minus sign appears in the answer (a consequence of the transition from –3 to –2) is also something I foul up sometimes. Oh well. Now onward.
The initial condition y(1)=2 allows us to pick out a particular solution by finding a value of C in the general solution:
Since y(1)=2, (3/5)15–(2/3)1–3+C=2. This means (3/5)–(2/3)+C=2 or C=61/15.

The solution is f(x)=(3/5)x5–(2/3)x–3+(61/15).

A different kind of differential equation
We look at dy/dt=ky where k is a constant. The independent variable is usually called t here instead of x. The reason for t (time) will become apparent, I hope. Notice that this is a very different kind of equation, because what's on the right-hand side is a constant multiplied by y. We can't solve it by just antidifferentiating ky because we don't know y. So a different approach must be used. There will be more about such equations in Math 152. This is a simple example designed to help you work with a number of useful applications.

Translation into English; what is this?
I asked people about an English language translation of the equation dy/dt=ky. This is difficult. Here is a possible candidate for such a translation:

The rate of change of y over time (dy/dt) is directly proportional to y itself. ("Directly proportional" means that when the amount of y changes, that the rate of change of y is changed by k mutliplied by the change.)
This is a terrible "translation". I am sorry. Maybe I'd better stick with adding fractions. English is too difficult!
There are many real phenomena which satisfy this sort of growth (or decay) rule. Examples include:

I'll try to discuss solutions and uses of this equation next time.


Tuesday, December 1 (Lecture #25)
Please think about the special offer for redoing a writeup.

FTC 1
If F´=f, then ∫abf(x) dx=F(x)|ab=F(b)–F(a).

Simple examples

  • 01x1/3dx=(3/4)x4/3|01=(4/3).
    Magic? I think it is very easy to not think about even "simple" computations like evaluating this integral. If we used Riemann sums, I believe getting an exact answer would require quite a bit of manipulation. Here all we need to do is guess (?) an antiderivative of x1/3 (hey, it comes from a power one unit higher, well, then we need to correct for the number that comes down in front, so the answer should be ...) and then the number practically presents itself. To me this is amazing technology.
  • 49(3x2–1/sqrt(x))2dx. There are various approaches which are possible. The simplest which occurs to me is just to expand the square. That is:
        (3x2–1/sqrt(x))2=(3x2)2+2·(3x2)·(–1/sqrt(x))+(–1/sqrt(x))2=9x4–6x3/2+1/x.
    So now we compute ∫499x4–6x3/2+1/x dx= (9/5)x5–6(2/5)x5/2+ln(x)|49=(9/5)95–6(2/5)95/2+ln(9)–((9/5)45–6(2/5)45/2+ln(4)). I would be content with this answer on an exam. Also, please note the big parentheses. I tell you with true humility that I have messed up such parentheses too many times. They are easy to forget.

    I wrote two "rules" for computations on our exams. They were:
    • Do no unnecessary simplifications. Do, however, find exact values of standard functions such as e0 and sin(Π/2).

    • Use lots of parentheses to prevent people (including yourself!) from misunderstanding your results.

  • 015ex+2e–xdx. Here the challenge in applying FTC 1 is getting an antiderivative. Well, ex is its own antiderivative. What about an antiderivative of e–x? First, I would "guess" e–x. But then I would quickly check and correct. That is, if we d/dx e–x, the result would be –e–x, so a suitable antiderivative would be –e–x. Now look:
    015ex+2e–xdx=5ex–4e–x|01= 5e1–4e–1(5e0–4e–0)=5e–4/e–1.

    FTC 2
    If F(x)=∫axf(t) dt, then F´(x)=f(x).

    An alphabet lesson
    What is ∫01x2dx? In the last lecture, we saw with several different techniques that this is 1/3. Then let's play a little logical game.

    The labels d(something) inside the definite integral do not get "communicated" to the outside. This is just like the index of summation in one of those ∑'s. This is the analogue of what is called a local variable in computer science, defined inside a for loop or a subprogram or subroutine. In math, one phrase used for such a thing is local variable. The letter is there to help the computation, but only has a meaning within the scope of the computation of the integral.

    Simple example
    Describe a function whose derivative is cos(x+ex).
    This is either very very easy, or, for reasons that will be discussed more in Math 152, very very hard. I will give the very very easy answer here.

    One function is F(x)=∫–3xcos(t+et) dt. This is a function whose derivative is cos(sqrt(x)+ex) by using FTC 2.

    Another function is G(x)=∫5xcos(w+ew) dw.

    Let me compare the two answers. One has inside variable t and the other has inside variable w. This doesn't matter. They both are "satisfactory" answers to the question asked because FTC 2 implies that the derivative of a definite integral with a variable upper parameter is the integrand's value at that upper parameter (hey, I'm using all of the high-priced words!) One answer is a definite integral from –3 to x and the other is a definite integral from 5 to x. Using property (*) from last time, the two answers are different by this: ∫–35cos(s+es) ds. What is this? It is a constant (I'm just using another variable, s, to again bring up the idea of a "dummy variable"). Two functions which differ by a constant have the same derivative. So that's o.k.

    By the way, it can be proved that no one can do much better in this problem than write the answer as a definite integral. There isn't any much simpler answer. Software can then plot this function, because people have spent a great deal of time and effort learning how to compute good numerical approximations to definite integrals very rapidly. The picture of F(x), the antiderivative of cos(x+ex) which is defined above, on the interval [0,2] was produced by Maple in less than three-tenths of a second. I am sure other software would do the job as well.

    More examples (a textbook problem)
    This is problem 23 in section 5.4. The graph of f is given to the right. The problem asks us to sketch a graph of A(x)=∫0xf(t) dt. I presume, since f's graph is given on the interval [0,4], that we are supposed to sketch a graph of A(x) on the same interval.

    There are various strategies for analyzing this sort of problem. Since the information is furnished graphically, I'd probably try to do the problem graphically, or at least begin it that way. I would think of the limits, 0 and x, and try to see how the definite integral (which I'd think of as signed area) varied as x varied.

    Let's consider first x's between 0 and 1. Here I think of a vertical line somewhere over the [0,1] interval, moving to the right. The area between that line and the y-axis, under y=2, is ∫0xf(t) dt. Since the line has height 2, the area is 2x, and the graph of A is a straight line segment starting from (0,0) with slope 2.
    When the vertical line passes x=1, we have accumulated 2 units of area, and A(1)=2. But now the "profile curve", f, changes height. It has height 1. We must add to the 2 units of accumulated area the new area we are getting between x and 1. Since the height of f is 1, and the base is x–1, we add on 1(x–1) units of area.

    The graph of A in this interval will be linear with a slope of 1, and it will start from (2,2).

    At x=2, we have accumulated 3 units of area, 2 over the interval [0,1], and 1 more over the interval [1,2]. Now x moves to the right in the interval [2,3]. The height of f is –1, so the definite integral will decrease: the geometric area is below the x-axis. We know A(2)=3 becuase of the accumulated area.

    But if x is between 2 and 3, the change in A is (–1)(x–2). So we need to show the graph of A decreasing, and it will be decreasing linearly in x, with a slope of –1.

    By the time the vertical line has gotten to x=3, it has accumulated 3–1 units of area. Yes, I know that 3–1=2, so actually A(3)=2, but somehow writing and thinking 3–1 helps me recall that + is assigned to areas above the x-axis and is assigned to areas below the x-axis.

    Between 3 and 4, the height of f is 0 (zero: nothing). There is therefore no change in the area as we move the upper limit on the definite intergral to the right. And therefore there is no change in A, and the graph of A is a horizontal line segment beginning at (3,2).

    From this we get a very good idea of the graph of A(x). It is 4 line segments with slopes of 2, 1, –1, and 0. It is a continuous function. The slopes of A correspond to the heights of f, and actually A is an antiderivative of f (the antiderivative with the initial condition A(0)=0), so f is the derivative of A. I think the graph looks like what's shown to the right.

    Part B
    Well, there's actually a part B) to this problem. The graph to the right is given, and, again, we are asked to graph A(x)=∫0xf(t) dt. Here the graph is a bit more complicated (well, it is more complicated to me: the other function had exactly 4 values, and this function has lots and lots of values!).
    I'll use a strategy different from what we did in A) to understand and solve this problem.
    Probably I would look at the graph and learn that A(0)=0 (there is no area from 0 to 0!). I would look a bit more and find that A(2)=1, because ∫02f(t) dt is the area of a triangle with base 2 and height 1, and (1/2)(2)(1)=1. Finally, I would see that A(4)=2, because there are two of those triangles. So I have the three dots shown to the right on the graph of A.

    Notice also that the graph of A is increasing because as we move the right limit to the right we are getting more and more area, and this is positive area since it is above the x-axis.

    What can we say about the derivative of A in the interval [0,2]? By FTC 2, this derivative is the function f, and the function f is just (1/2)x. That means the second derivative of A is 1/2, and this is positive. So the graph of A between 0 and 2 is concave up, and connects (0,0) and (2,1).

    I actually know a formula for A, since A(0)=0 and A´(x)=(1/2)x. It must be (I antidifferentiate and get the correct constant!) A(x)=(1/4)x2. So what I see between 0 and 2 is a piece of a parabola.

    Between 2 and 4, the function keeps increasing. But the slope of the function, A´, is positive: it seems to be a line segment from (2,1) to (4,0). But the derivative of that is –1/2, so the second derivative of A is negative. A is increasing and concave down. I think the graph is must look like what is displayed to the right.

    In fact, between 2 and 4, A´(x)=–(1/2)x+2. I also know that I have accumulated 1 unit of area by the time we get to 2, so that A(2)=1. I can solve this initial value problem: antidifferentiate to get A(x)=–(1/4)x2+2x+C. Use A(2)=1 to get C: –(1/4)22+2(2)+C=1, so C=–2, and the formula for A(x) in this interval is –(1/4)x2+2x–2. We can check this by plugging in x=4, so the result is –(1/4)42+2(4)–2=–4+2(4)–2=2 which is correct.

    Another thing which is not obvious is that the function A is differentiable. Since A is defined by two nice formulas, it isn't very surprising that A´ exists inside the two halves of the domain. In fact, A is differentiable at x=2, also. This can be checked with effort by looking at the difference quotient (there is a similar check in the workshop problem solution inserted in the Rutgers edition of the textbook). But I am willing to believe that A´(2) exists, because:

    • If A(x)=(1/4)x2 then A´(x)=(1/2)x so A´(2) should be 1.
    • If A(x)=–(1/4)x2+2x–2 then A´(x)=–(1/2)x+2 so A´(2) should be 1.
    I bet that the slope of the tangent line at 2 is 1.

    Movement of a particle
    Let's assume that the velocity of a particle traveling on the x-axis is given as a function of time: v(t)=4t2–t4 for t between 0 and 5. (As far as I know, this is a rather non-physical example, but I hope that the questions and ideas contributing to the solutions are correct and useful.)

    1. For which values of t is the particle traveling to the right? To the left?
      "Travel to the right" in terms of velocity means "v(t)>0" and of course "to the left" is "v(t)<0". Since v(t)=4t2–t4=t2(4–t2). Notice that t2 is non-negative, so that 4–t2 has the sign information. We should consider 4–t2 on the time interval [0,5]. Of course, 4–t2=0 when t is +/–2. So (considering everything) v(t)>0 when t is between 0 and 2 and v(t)<0 when t is between 2 and 5. Notice that velocity is 0 when t=0 and t=+/–2.
    I had to stop here since I ran out of time. I will continue with this next time. I want to investigate and contrast the ideas of displacement and distance traveled. This might be interesting to physics people.


    Tuesday, November 24 (Lecture #24)
    The definite integral (a quick review)
    Suppose some more general function f is defined on an interval [a,b], and suppose either that f is continuous or piecewise continuous (that is, continuous except for a few jumps). Then as the maximum length of the subintervals used in the Riemann sums for this function get smaller, the values of the Riemann sums will approach a unique number which is called the definite integral of f on the interval [a,b], and the notation used is ∫abf(x)dx.
    Here a is called the lower limit and b is called the upper limit. The integral sign, ∫, is a sort of stylized abbreviation of the word Sum. The function f(x) is sometimes called the integrand. The "dx" is there to remind you of the length of little subintervals.

    Last time we saw that if we had a function f defined on an interval [a,b], then we could introduce a rather elaborate collection of objects which are used to analyze f's behavior on all of [a,b].

    If [a,b] has a partition P (which break up [a,b] into a number of subintervals) and a collection of sample points (the text's intermediate points) C, then we defined the Riemann sum of f using P and C:
    R(f,P,C)=∑j=1nf(cj)Δxj.
    If the length of the longest subinterval→0, then (for the functions we will consider in this course!) the Riemann sums approach a limit which is called the definite integral of f from a to b and written ∫abf(x)dx.

    Definite integrals have some very useful properties which people use frequently.

    Property (*) and consequences
    Suppose a<b<c. Then ∫abf(x)dx+∫bcf(x)dx=∫acf(x)dx and I will call this statement (*). I am happy if you believe this statement. I hope that the picture to the right is enough verification. The picture has red for area below the horizontal axis and green for area above the axis because I wanted to remind you how area is counted for the definite integral. It is possible to prove this statement from the definition with Riemann sums.

    Unexpected consequences of (*)
    While most people are willing to believe (*), there are some results which may take you some time to learn to use. For example, in the equation
        ∫abf(x)dx+∫bcf(x)dx=∫acf(x)dx
    what if we change b to a? Then we seem to get
        ∫aaf(x)dx+∫acf(x)dx=∫acf(x)dx
    and the only way this could be true is if
        ∫aaf(x)dx=0.
    Most people find this easy enough to believe: this quantity is the area of a "region" with width equal to zero, and some height, and such an area should be 0. So any definite integral whose upper and lower limits are the same must be 0.

    O.k., if you believe that one, then I will make another change in the equation:
        ∫abf(x)dx+∫bcf(x)dx=∫acf(x)dx.
    Change c to a in this equation, and use the fact that the right-hand side will become ∫aaf(x)dx which we already "know" is 0. So we get:
        ∫abf(x)dx+∫baf(x)dx=0
    and this certainly means that ∫baf(x)dx=abf(x)dx
    so that when the limits of a definite integral are interchanged, then the sign of the integral is changed! (What we are in fact computed is called an oriented area, and more complicated than what I've told you so far.)

    Example(s)
    Suppose f is the function whose graph is shown to the right. It is piecewise linear, and made of four line segments. Then all of the values of these integrals:
        ∫02f(x)dx    ∫–11f(x)dx    ∫3–1f(x)dx    ∫44f(x)dx
        ∫13f(x)dx    ∫–12f(x)dx    ∫30f(x)dx    ∫0–1f(x)dx
    are contained in this collection of numbers:
        –1    0    1/2    1    3/2.
    Can you match each definite integral with one of these values?

    Property (**)
    This is extremely useful when you are confused and need to make some sort of estimate. Suppose, somehow, you know that all of the values of f(x) on the interval [a,b] are between m and M. That is, you know m≤f(x)≤M for all of those x's. Then (look at the graph!) the "area" (actually the definite integral) will be trapped inside a box. So the result is:
        m(b–a)≤∫abf(x)dx≤M(b–a).
    This is because the wiggly region contains the smaller box and is contained by the larger box. It looks silly, but the estimates can really be useful in checking computations.

    I mentioned that definite integrals are used to compute many quantities of interest to engineers, includiong such things as work, power, energy, pressure, volume, length, etc. This may not be too clear right now, but a number of these are discussed in the next course (Math 152). The key is that for all of these quantities, (*) and (**) are true in some way.

    Yet another example ...
    Here is a final example of how to compute an area (or a definite integral) in the most direct fashion: by getting the area of a collection of approximating rectangles, and then analyzing what happens as the partition gets "finer".

    The example is the area under y=x2 from 0 to 1. This means the area enclosed by the x-axis, y=1, and y=x2. It is an area in the first quadrant. We will approximate the area, which is the definite integral ∫01x2dx by a collection of Riemann sums. Here f(x)=x2.

    1. Partition [0,1] into N equal subintervals, where you should think that N is some large positive integer. Since the original interval has length 1, each subinterval will have length 1/N. The actual partition points will begin 0/N, 1/N, 2/N, 3/N, 4/N ... and will end with (N–1)/N and N/N.
    2. Get sample or intermediate points. In this computation, I will choose the right-hand endpoints of each subinterval as the sample points. So in the first subinterval [0/N,1/N], the sample point will be 1/N. In the second subinterval [1/N,2/N], the sample point will be 2/N. In the third subinterval [2/N,3/N], the sample point will be 3/N. ... In the Nth subinterval [(N–1)/N.N/N], the sample point will be N/N.
    3. Compute the function values at the selected points. These numbers are the heights of the approximating rectangles, and they are f(1/N)=12/N2 and f(2/N)=22/N2 and f(3/N)=32/N2 and ... f((N–1)/N)=(N–1)2/N2 and f(N/N)=N2/N2. Wow!
    4. Multiply each of the heights by the widths of the rectangles. Each of the widths is identical, however, so things aren't so bad. The results for the areas of the rectangles are:
      12/N2·(1/N) and 22/N2·(1/N) and 32/N2·(1/N) and ... (N–1)2/N2·(1/N) and N2/N2·(1/N). More wow!
    5. Add 'em up. So this is:
      12/N2·(1/N)+22/N2·(1/N)+32/N2·(1/N)+...(N–1)2/N2·(1/N)+N2/N2·(1/N).
      More compactly written, the Riemann sum is ∑j=1Nj2/N3. I combined the powers of N "downstairs". You could also pull out the 1/N3 because it multiplies every term in the sum, and then the result would be (1/N3)∑j=1Nj2.

    N=5
    If we had N=5, the approximation would be (1/53)(1+4+9+16+25) which is (1/125)(55). The mysterious part is the 55, of course.

    A magic formula
    Well, everyone knows (it is in the book on page 317, so ... it is possible to know it!) that ∑j=1Nj2 is (1/3)N3+(1/2)N2+(1/6)N.
    We actually checked that if N=5 is plugged into this formula, the result is 55. That's nice. So maybe the formula is correct.

    Getting the exact value of the definite integral
    Since we know that ∑j=1Nj2 is (1/3)N3+(1/2)N2+(1/6)N and that the Riemann sum is this divided by N3, then the value of the Riemann sum is (1/N3)((1/3)N3+(1/2)N2+(1/6)N) which is (1/3)+(1/2N)+(1/6N2) and certainly as N→∞, this must approach 1/3. So ∫01x2dx=1/3.

    Change the problem: make it harder!!!
    These techniques seem to be (they are!) very intricate. Let me show you how to solve this problem by first making it more difficult. The people who discovered that the (seemingly) more difficult problem can be solved very neatly really had a lot of intellectual courage.

    So instead of studying ∫01x2dx, we will study the function
        A(b)=∫0bx2dx.
    We really want to know A(1), but we'll consider this more general problem anyway.

    What do we know?
    Well, there is one value of the function that is extremely easy to compute: A(0)=∫00x2dx=0.

    That seems to be the only simple thing. But this is a CALCULUS course, and maybe we should try to differentiate A(b). This, which certainly should seem almost ludicrous, turns out to be the successful approach.

    In order to find A´(b) we will need to consider A(b+h)–A(b) and then divide the result by h.
    Certainly (*) implies that
        ∫0bx2dx+∫bb+hx2dx=∫0b+hx2dx
    so that A(b+h)–A(b) is ∫bb+hx2dx.

    But we can use (**) now. f(x)=x2 is increasing on [b,b+h], so for all x in the interval, b2≤f(x)≤(b+h)2. So, with m=b2 and M=(b+h)2, we see that
        b2·h≤A(b+h)–A(b)≤(b+h)2·h
    since h is the width of the interval. Now divide by h:
        b2≤[A(b+h)–A(b)]/h≤(b+h)2.
    Notice that both estimates →b2 as h→0. This means that A(b) is differentiable, and that A´(b)=b2.

    An initial value problem
    We have a function, A(b), with the following properties:
        A(0)=0 (initial condition);
        A´(b)=b2 (differential equation).
    This is an initial value problem. All antiderivatives of b2 have the formula (1/3)b3+C. If A(b)=(1/3)b3+C then A(0) must be C, but the initial condition states that A(0)=0, so C=0. Therefore, A(b)=(1/3)b3.

    Now solve the original problem
    Since A(b)=(1/3)b3, then A(1)=(1/3). No problem!

    Hey, things are easier than they look
    In fact it turns out that we didn't need to figure out the C, because the value we need to compute the integral from 0 to 1 is A(1)–A(0), and any antiderivative will give the same result (the "+C" adds and subtracts, and doesn't change the answer).

    The Fundamental Theorem of Calculus, version 1
    If F´=f, then ∫abf(x)dx=F(b)–F(a).
    In fact, the notation usually used "abbreviates" F(b)–F(a) by F(x)|x=ax=b. And "Fundamental Theorem of Calculus" is also usually abbreviated as FTC.

    Here is how the original area desired would be computed.

    1. Let's find the area under y=x2 from 0 to 1.
    2. Recognize that this is the same as computing ∫01x2dx.
    3. We know an antiderivative of x2: it is (1/3)x3.
    4. FTC then declares that the definite integral (1/3)x3|x=0x=1 and this is (1/3), the value of the area desired.

    The area under a bump of sine
    What is the area undo a bump of sine? Well, first think a bit: the idea of bump maybe means the area between two places where sine is 0 and where the curve is above the x-axis. So the area would be computed by the definite integral ∫0Pisin(x) dx. Usually having some estimate of the size of a quantity, even before computing, is a good idea. Otherwise you may make some simple error, and get some result which is a huge mistake.

    One overestimate of this definite integral is obtained by looking at a rectangular box which contains the area. The width of the box is Π and its height is 1, so the integral should be less than Π·1=Π. An underestimate of the area could be gotten by looking at two triangles under the graph. Each has base Π/2 and height 1, so the total area of the two triangles is 2·(1/2)·(Π/2)·1=Π/2 (the 1/2 comes from the formula for the area of a triangle). One student suggested 0 as an underestimate. While that is certainly valid, I like under- and overestimates which are closer to the exact answer (as long as I don't have to work very hard!). Now we know
    Π/2≤∫0Πsin(x) dx≤Π.

    To compute this area exactly, we use FTC. We need an antiderivative of sine, and that happens to be –cos(x). So:
    0Πsin(x) dx=–cos(x)|0Π=–cos(Π)–(–cos(0))=–(–1)–(–1)=2. The (potential!) confusion of minus signs occurs often when applying FTC, so some care is needed. The exact area is 2, and this is certainly between Π/2 and Π.

    A fake computation: the "area" under a bump of cosine
    I then tried to rush ahead and compute the "area" under a bump of cosine. Or, rather, I compute this definite integral: ∫0Πcos(x) dx. I asserted that the result should be ... well, should be ... something.

    Here is the computation using FTC, since an antiderivative of cosine is sine.
    0Πcos(x) dx=sin(x)|0Π=sin(Π)–(sin(0))=0–0=0.
    What happened to the "area" under the bump? We computed a definite integral, and that's the limit of Riemann sums, and they count geometric area under the x-axis negatively. The graph of cosine on the interval [0,Π] exactly balances the positive over-the-axis region with a negative under-the-axis region. So it is 0 both computationally and geometrically!

    Another bump but negative
    I asked students what the curve y=x2(x–1) from 0 to 1 looked like. After a while we got a picture similar to what is shown to the right. I asked what the area was inside the bump. Please observe that the function is negative, so the definite integral would be negative, but the area would be positive.

    A solution might go like this:
    01x2(x–1)dx=∫01x3–x2dx=(using FTC)=(1/4)x4=(1/3)x3|01=(1/4)–(1/3)=–1/12.
    Therefore the area is 1/12.

    Return of the second exam
    I returned the second exam. Information about grading and statistics concerning the results are here. If you think there's been a mistake made with the grading of your exam (an almost certain event, considering the number of problems, the size of the class, and the too fallible human who did the grading), please check the grading guide, and then bring the exam to me.

    I note that the median grade, which I regard as probably the most important number statistically for such happenings, is up from the first exam median.

    Course grades will be strongly influenced by both your own efforts and also how the class as a whole does on the Math 151 final. Therefore the final exam results will be used to calibrate or measure or compare performance of the 153 students with the overall 151 group's work, and I will adjust course grades appropriately. I mean that the group achievement will be considered, so it is in the interest of individual members of the class that the whole class do as well as possible!


    Thursday, November 19 (Lecture #23)
    The second exam was given.


    Tuesday, November 17 (Lecture #22)
    Algebraic formulas and human beings
    We developed a rather involved technique last time to compute certain areas. For example, we needed to know that 1+2+3+4+...+n is also given by (1/2)(n+1)2-(1/2)n-1/2. Then we could find the area "under" y=x. A similar formula:
                          3          2
                   (n + 1)    (n + 1)
    12+22+...+n2 = -------- - -------- + n/6 + 1/6
                      3          2
    
    which was known in many cultures allows the computation of areas under y=x2. We could get even more. For the sum of the first n seventh powers (that is, 17+27+...+n7) the result is
                      8          7            6            4          2
               (n + 1)    (n + 1)    7 (n + 1)    7 (n + 1)    (n + 1)
               -------- - -------- + ---------- - ---------- + --------
                  8          2           12           24          12
    
    In a number of cultures, this sort of information was treated almost as a secret, and the details and use were forgotten. Similar, even more complicated results would allow areas of the trig functions and even certain exponentials to be computed. But ... there's a better way. But first I want to convince you that the sort of problem we're looking at is worth thinking about. Most engineers, once they're out of calculus courses, never compute an area again. But they do compute other things. Let me show you an example of an "other thing".
    A question of blood...
    I asked how we could estimate how much blood the heart pumps out to the body, drawing an especially silly picture on the board (look left). Knowing the quantity of (hopefully!) oxygenated blood which is pushed around the body can be a useful part of health assessment. How can we measure the blood flow through the aorta, the major artery emerging from the heart? A better diagram of a heart is to the left. The heart is a very complicated and wonderful pump.

    The Aztec solution
    One method might be to take a knife, preferably an obsidian knife, and rapidly cut through the chest wall, sever the aorta, and cause the blood that would flow through it to fill up a bucket. In about five minutes we'd probably have a good idea of how much blood flows through the aorta. There are several criticisms possible of this "protocol". One criticism might be that, well, maybe the person involved might not be good for much afterwards. Another criticism could be that this would not show the actual pumping capacity of the aorta, because maybe within a minute or two some ... difficulties would occur. In some sense, this Aztec protocol is the ultimate in what might be called destructive testing. By the way, obsidian is "a dark glassy volcanic rock formed from hardened lava" and was actually used for certain similar purposes in that culture.

    Another solution: cardiac catheterization
    The American Heart Association and the NIH (National Institutes of Health) both have lots of information about this. Here are some quotes from these sources:

    Very much simplified ...
    Imagine a thin plastic coated wire inserted into your circulatory system and threaded up into the aorta. At the end of the wire is a sort of microscopic windmill: a bunch of little fans which spin as the blood flows. As they spin, they move a bunch of wire around a magnet. Since magnetism+movement+wires=current ( a wonderful fact!), electric current is created and passed back through the wire into a measuring device. This is certainly very much simplified. Now I will further simplify and imagine some numbers. This is not reality, but a math course! The following numbers are invented, and the whole purpose of this "exercise" is to get you to think about what's going on.

    Elapsed timeFluid flow rate
    1 min30 cm3/min
    2 min20 cm3/min
    4 min40 cm3/min
    5 min25 cm3/min
    The blood flow will be given in cm3/min (cubic centimeters per minute) and the time will be measured in minutes since the beginning of the procedure. The information learned is displayed to the right.

    This isn't much information. But how can we use what we've learned? A standard way to use these numbers is the following. It is certainly very approximate in this case.

    At the end of the first minute, we're told that the flow is 30 cm3/min. Hey, we could estimate how much blood is pumped by making the assumption that the flow rate is constant during the time interval involved. Then we see that 30 cm3/min·1 min=30 cm3 (flow rate multiplied by duration) would be pumped. I am not asserting that this is necessarily correct. It is an estimation. We are using the information, and the method is very simple. Similarly, during the second minute, we can estimate that 20 cm3/min·1 min=20 cm3 is pumped. The flow rate changed during the second minute. We're faced with a slightly different "challenge" in using the third line of the table. It has elapsed time (since the start of the procedure) as 4 minutes. So the duration has now changed from 1 minute to 2 minutes. (Why? Well, y'know, in the real world things maybe don't work as you'd like. Maybe the machine broke, or the technician didn't pay attention, or ... in any case, we need to work with the data we have!) So during those two minutes, we have (flow rate·duration) an estimate of 40 cm3/min·2 min=80 cm3 blood pumped. Finally, in the last minute of this procedure, we estimate that 25 cm3/min·1 min=25 cm3.

    Let me abbreviate just a bit. The units are repetitive and I will omit them (this is a math course!). So here are the essential details of the computation:
        30·1+20·1+40·2+25·1=155.
    That's what we're computing.

    If we wanted a picture of the computation, maybe what is shown to the right gives you some idea. The dots represent the actual known data. The area of the rectangles shown is given by the product of the appropriate flow rate and duration. Yes, there are other possible approximations using the data given. We will study some of them in Math 152, but here only a rather simple-minded approach will be used.

    More information
    Elapsed timeFluid flow rate
    30 sec32 cm3/min
    1 min30 cm3/min
    1 min 30 sec28 cm3/min
    2 min20 cm3/min
    2 min 30 sec–6 cm3/min
    3 min–8 cm3/min
    3 min 30 sec34 cm3/min
    4 min40 cm3/min
    4 min 30 sec43 cm3/min
    5 min25 cm3/min
    Now let's refine the data. Let me suppose that we have no "technical" problems, and that we are recording the flow rate every 30 seconds. We could imagine the numbers given in the table to the right. Please notice that the data we've already obtained is part of this table -- I am imagining that we're measuring the same process as before, only we are sampling things more often. Here we have readings every thirty seconds with no data points are omitted. So we actually have 10 pairs of numbers.

    There is another complication which has occurred, and I hope you notice it. Some of the flow rates (2 min 30 sec and 3 min)are negative. Could this happen? Well, actually maybe yes. The heart is a terrific pump but maybe sometimes things don't always synchronize well or work perfectly: maybe the heart valves don't close all the way or don't close at precisely the correct time in the pumping cycle, etc. This complicates our task. We could ask what the heck we want to measure in this situation, and several answers are possible. We might want to measure the total pumping effort of the heart. Or we might want to know the net amount of oxygenated blood which is pushed out into the body. Let me look at the second quantity. In that case, the negative flow rates affect the total blood pumped because, indeed, that amount in that time should be subtracted, just like the sign of the flow rate indicates, from the total.

    What should be computed here? If we do flow rate multiplied by duration, we should be consistent (the units!) and realize that, for example, 3 min 30 sec is 3.5 minutes. So the numerical computation here is
        32·.5+30·.5+28·.5+20·.5+(–6)·.5+(–8)·.5+34·.5+34·.5+40·.5+43·.5+25·.5=119.
    And this estimate, using this data, is 119 cm3. (Ms. O'Sullivan asserts that the total is 109. Hah!) This is much less than the earlier figure, I guess because the data at 4 minutes over a duration of 2 minutes led to a big overestimate.

    A picture of what we computed is to the right, which I sort of laid over the old bar chart whose outlines where visible are indicated with dashed lines. I put in the new data points, and then the computation above is indicated by the areas of these rectangles. But the total area of the rectangles which are below the horizontal axis is subtracted from the total. Can you "see" the change from 155 to 119?

     
    Ideally ...
    You might imagine that ideally we make lots and lots of measurements (in fact, this is more like what is done, because I think there is a computer recording the flow rates very frequently and then making the computations that I'm discussing in such a silly way). What should we think about? We could have lots and lots and lots of data points. And then the rectangles get very thin. And then the data points look very much like a curve. The number that's wanted is the total blood pumped out to the body, but the picture which might result could look a lot like what is shown here. I have again overlaid it with the rectangles from the previous discussion. So it seems there might be some kind of flow rate curve, and that the blood pumped out to the body, the net blood out, is the area of the colored regions above the curve minus the area of the colored region below the curve.

    I hope that you can see some resemblance between this and the discussion of the area of the hand which we did previously. Here we are approximating quantities of liquid, and there we were trying to approximate areas. It turns out that this computational idea can be applied in many situations, and it is used in essentially all fields of engineering and applied science. I now need to tell you the official vocabulary.

    Mass from density measurements
    I very briefly mentioned another almost realistic setup. We might have a bar with varying densities whose total mass we want to estimate. A sort of picture is to the right, with the different colors used to imply different densities. Direct measurement (lifting the bar, for example) might not be feasible. We could imagine taking small samples of the bar's materail at several points, and then measuring the densities of these samples. We could then multiply the density by the appropriate length of the bar and take the sum of these quantities as an estimate of the bar's mass. This would be a situation whose mathematical outline would be very similar to the blood flow analysis we've just done.

    Vocabulary
    Start with the following: a function f defined on an interval [a,b].

    What this means geometrically ...
    To the right is my attempt to show you what a Riemann sum might look like. But I've made it quite simple. There are only seven subintervals, and the curve which is the graph of f(x) is also not too complicated. The dots on the x-axis denote the sample or intermediate points, and the vertical lines above them are the lengths f(cj). The boxes have areas which are equal to the pieces of the Riemann sum, f(cj)δxj. The f(cj) is the height and the Δxj is the width of a rectangle.
    An example
    To the left is a graph of a function, y=f(x), where f is defined on the interval [0,5]. f's graph is two line segments, the first horizontal from (0,1) to (2,1) and the second tilted from (2,1) to (3,0), followed by the lower half of a circle of radius 1 centered at (4,0), a circular arc going from (3,0) to (4,–1) to (5,0).
    Here are some partitions, choices of sample points, and the resulting Riemann sums.
    Partition 0, 2, 3, 5.
    Sample points 1, 3, 4.
    The function values at the sample points are:
    f(1)=1, f(3)=0, f(4)=–1.
    Riemann sum 1(2–0)+0(3–2)+(–1)(5–3)=0.
    Choosing the sample point at 3 gives a zero contribution to the Riemann sum.
     
    Partition 0, 2, 3.5, 5.
    Sample points 1, 3, 4.
    The function values at the sample points are:
    f(1)=1, f(3)=.5, f(4)=–1.
    Riemann sum 1(2–0)+.5(3.5–2)+(–1)(5–3.5)=1.25.
    This was the QotD!
    Partition 0, 2, 3.5, 4.5, 5.
    Sample points 2, 2, 4, 5.
    The function values at the sample points are:
    f(2)=1, f(2)=1, f(4)=–1, f(5)=0.
    Riemann sum 1(2–0)+1(3.5–2)+(–1)(4.5–3.5)+0(5–4.5)=2.5.
    The definition allows the sample point to be on the boundary of the subinterval, so two adjoining subintervals could possibly share their sample points. This is a bit wasteful in terms of information, though.
    Also notice that the rectangle on the rightmost interval has height 0.
     
    Partition 0, 1, 2, 3, 4, 5.
    Sample points .7, 1.44, 2.5, 3.5, 4.75.
    The function values at the sample points are:
    f(.7)=1, f(1.44)=1, f(2.5)=.5, f(3.5)=–sqrt(3/4)≈–.866, f(4.75)=–sqrt(7/16)≈–.661.
    (The numbers for f's values at 3.5 and 4.75 come from the equation of the lower unit semicircle, –sqrt(1–x2), suitably translated to the right.)
    Riemann sum 1(1–0)+1(2–1)+.5(3–2)+(–.866)(4–3)+(–.661)(5–4)=.973.
    Notice that putting another sample inside the interval from 0 to 2 didn't improve the approximation very much. If taking samples is difficult or expensive (and it might be in real life!) then probably increasing the sampling where functions wiggle a lot rather than where the values are (relatively) constant is a good idea. This is, in fact, what the approximation strategies inside most calculators try to do.

    Part of the impression I would like to leave with you, even with just a few examples, is that Riemann sums can be quite complicated objects. They can be irritating to compute, and there are so many of them that understanding them may be difficult.


    Thursday, November 12 (Lecture #21)
    We are about to jump into something that will seem very different and which is computationally and conceptually quite involved. But there will be a remarkable connection between this topic and what we have already studied which will be revealed in a few lectures. In this one lecture, we investigate a topic that was the object of centuries of inquiry in a number of cultures. I will begin with a seemingly silly question.

    Hand area
    What is the area of my hand? There was, of course, some laughter. But the question and other similar inquiries, while superficially silly, is actually quite important. Appropriate drug doses are sometimes calculated in terms of BSA, body surface area. This is especially important when prescribing drugs for infants or drugs with strong side-effects (for example, some drugs used in cancer treatment). And knowledge of body surface area is important in trying to understand heat balance when people exercise. There are many formulas for BSA in addition to the Dubois and Dubois formula which was mentioned in an earlier workshop problem.

    So my question is much more modest: how can we compute the area of my hand? We discussed this. There are many methods, but if you think about area, there are really not very many shapes whose area we can compute exactly. The most important and simplest shape is a rectangle, where we know the area is the product of the length and width. We should try to take advantage of this. So look at an image of my hand, and ... well, after discussion we came up with this:

    1. Superimpose a square grid over an image of my hand.
    2. Put boxes inside.
    3. Put boxes outside.
    4. Count the boxes totally inside. Count the boxes which contain any (even little!) piece of the hand image (these are the "outside" boxes).
    5. Compute the area of the inside boxes and the area of the "outside" boxes.
    6. The hand area will be between the inside box area and the outside box area.
    Let me "implement" (!) this and get some information.

    I'll suppose first that I put the image of the hand in a square grid which is, say, 1 mm (millimeter) on a side. I don't know if this is physically too reasonable, but, what the heck, I could get something that looks like the picture to the right.

    Then I could count the grid boxes which are entirely within the hand image. There was some objection to this, because, you see, I could always be a little clever (?) and maybe sort of count some boxes which are almost entirely in and then compensate by dropping the count by 1 if I count a bunch of those.

    I commented that I wanted a process (an algorithm) which could be done totally "mechanically", leaving nothing to choice, and which was extremely simple-minded.
    I'll suppose first that I put the image of the hand in a square grid which is, say, 1 mm (millimeter) on a side. I don't know if this is physically too reasonable, but, what the heck, I could get something that looks like the picture to the right.

    I think if I count correctly there are a total of 18 boxes entirely inside the hand image.

    Now for an overestimate. Let me start with the boxes which are entirely inside, and then add on any boxes that have even the slightest "tinge" (?) or hint of the hand image inside. Again, maybe there are objections: we could be cleverer (?) and sort of estimate how much of a box is inside, etc. But then we would have to make decisions, and some of those decisions maybe could be difficult. Here, if we have a simple-minded scheme, I'll bet that any two people will come up with the same answer. At least we get a result that's stable, and probably doesn't depend on the observer too much.

    There are 18 all-inside boxes, and (if I count correctly!) 60 more boxes are needed to cover all parts of the hand image. The total of the outside boxes is therefore 78.

    The result?
    If I believe that area of bigger shapes is bigger, then I see that the area of the hand is between 18 mm2 and 78 mm2. And there is a huge gap between these over- and underestimates of 60 mm2.

    More information?
    How can we improve this? That is, how can we get over- and underestimates which are closer to the actual area of the hand, so the gap will be smaller? We discussed this. The idea of improving the estimates is not totally obvious, and the whole procedure that's being discussed in this lecture evolved over many centuries in at least 4 different cultures (in China, Greece, India, and perhaps also in Egypt and associated cultures). Lots of smart people thought about it.

    We get try a grid with more subdivisions: this is called refining the grid. We could just halve the size of the square sides. Maybe the result is something like what is shown to the right.

    Now the idea is maybe not so easy to implement, at least for human beings whose eyesight and counting ability is not so perfect. But we can try anyway.

    Well, if I have counted correctly, there are 63 new squares which are completely inside the hand image. This is in addition to the 18 old squares. But, wait: the new squares have sides which are half the length of the old squares. So each of the new squares has one quarter the area of the old squares. So actually we have now succeeding in underestimating the area by 18+(63/4)=25 and 7/8.

    Now let's get an overestimate. We need to add on all of the little squares which contain any part of the hand image. If I have counted correctly (!!) there are 118 of them. That means we need to add one quarter multiplied to 118 to the previous underestimate in order to get the corresponding overestimate. So 118/4 is 29 and a half. Add this on to the previous 25 and 7/8, and the overestimate is 45 and 3/8.

    Underestimate, overestimate, and gap
    Let me summarize what we have done:

     UnderestimateOverestimateGap
    1-by-1 squares187860
    1/4-by-1/4 squares25.87545.37529.5
    And so on ...

    The secret is that this sort of approach is just about all we can do with real physical quantities. You can really only make better (or worse!) estimates: over- or underestimates. The ideas outlined here, taking finer and finer grids, will lead to overestimates which shrink and underestimates which increase, so that (we hope!) the gap between them →0: refine until you get as close as you want. And this is about all we can do. I don't think the situation is hopeless, but I do think it is compicated.

    Attribution
    The neat idea of asking for the area of "my hand" is something I borrowed from Professor J. Rosenstein of our math department. I think it is clever. It is a question which brings up important ideas immediately and clearly.

    An exact computation
    We discussed if we could do an exact computation, not just an estimate, and this I was happy to try, since many people seemed rather distressed about the statements made previously. So I will present a simple example of a general idea. The general idea is very successful, although the simple example may seem to be overelaborate. I will try to compute the exact area "under" y=x for the interval [0,1]. (Yes, I am aware that we know this area -- the ideas are what is important -- as soon as we get the ideas we'll be able to do much more complicated computations.)

    How to begin
    We will try to get over- and underestimates of the area We could divide the interval [0,1] into 4 equal parts using 0 and 1/4 and 2/4 and 3/4 and 1, which I'll choose to call 4/4.

    Then put rectangles underneath y=x in each subinterval. We will be slightly economical and put the largest rectangles we can under the y=x, but we will be slightly simple-minded and only look at rectangles which have sides parallel to the coordinate axes. Then we compute the area of these rectangles:
      0     1      1     1      2     1      3     1  
     --- · --- +  --- · --- +  --- · --- +  --- · --- 
      4     4      4     4      4     4      4     4
    
    The 1/4's come from the width of the rectangles, and the varying heights (even including the somewhat absurd 0/4) from the heights of the rectangles.
    Now we need an overestimate, and the true area will be "captured" between these two estimates. Again, we will use the interval divided into 4 equal pieces, and rectangles with sides parallel to the coordinate axes which just contain the region of interest to us. So the area of these rectanges is:
      1     1      2     1      3     1      4     1  
     --- · --- +  --- · --- +  --- · --- +  --- · --- 
      4     4      4     4      4     4      4     4
    
    Finally, if we want to see how much gap or error there is between these over- and underestimates, we can look at the area that is inside the big rectangles and outside the small rectangles. The result (because y=x is so simple!) is 4 squares, each with side length 1/4. So the gap is 4(1/4)(1/4)=1/4.

    The whole setup
    We could do something similar with other numbers of subdivisions of [0,1]. Here are some numerical results:

    # of
    subdivisions
    UnderestimateOverestimateGap
    4.37500 .62500.25000
    10.45000 .55000.10000
    25.48000 .52000.04000
    100.49500 .50500.01000

    What really happens, in general
    Here it helps to get a little bit abstract. Suppose we divide the interval [0,1] into n equal pieces, where you should think that n is some large integer. Then the underestimate will have total area (using heights over the left-hand endpoint of each subinterval shown in the diagram to the right)

      0     1      1     1      2     1      3     1          n-1    1
     --- · --- +  --- · --- +  --- · --- +  --- · --- + ··· + --- · --- 
      n     n      n     n      n     n      n     n           n     n
    
    In fact, we can "massage" this underestimate a bit and get for the underestimate:
        (1+2+3+...+(n–1))·(1/n2).

    The overestimate uses the heights over the right-hand endpoint of each subinterval, and then the overestimate is:
        (1+2+3+...+n)·(1/n2).

    The gap between the over- and underestimates is n·(1/n2)=1/n.

    So what happens as n→∞? Since the gap is 1/n, this will →0. So let me look at the overestimate. This is (1+2+3+...+n)·(1/n2). Let's get a formula for 1+2+3+...+n.

    A formula
    You may remember the trick (maybe done in high school when you studied arithmetic progressions). The trick is writing two versions of the sum in oppositite order. Look:

     1 +   2   +   3   +   4   + ... + (n-1) + n 
     n + (n-1) + (n-2) + (n-3) + ... +   2   + 1
    Now add the sums columnwise. So we get 1+n which is n+1 and 2+(n–1) which is n+1 and ... all of the column sums are the same (that's right because the top is going up by 1 each step while the bottom is going down by 1 each step). There are n of the n+1's, so the total of all of the numbers is n(n+1). But this is two copies of the sum we want, so we need to divide by 2.

    I think we have verified that 1+2+3+...+(n–1)+n is [n(n+1]/2. And the overestimate of the area, is this divided by n2. So a formula for the overestimate is [n(n+1]/[2n2]. What happens to this as n→∞? Well, we have studied such limits, and we could either divide top and bottom by n2 or we could use L'H twice. In any case, the limit will be 1/2.

    A spectacular result?
    I have now proved (done an exact computation!) that the area of the triangle is 1/2. And I guess I knew that. But this whole approach is a template for more complicated reasoning.

    Notation
    We will need some notation. Lots of sums occur when we do area computations, and also many other computations. So a special way of writing these sums which takes less space has been developed. The Greek capital letter sigma, ∑, is used. Here are some examples:

    Examples of summation notation
    Written outAbbreviated
    1+2+3+4+5j=15j
    32+42+52+62j=36j2
    {1/2}+{1/4}+{1/6}+{1/8}j=14{1/[2j]}
    1–2+3–4+5j=15(–1)j+1j

    The notation, once you get used to it, is wonderfully economical. I should remark that the summation index, which is j in all of the examples above, doesn't really matter. That is ∑j=13j4 and ∑k=13k4 both mean the same thing, which is 14+24+34.

    How to approximate area
    Suppose that we have a function, f, defined on an interval [a,b] and we want to compute the area under y=f(x). That's the area bounded by the x-axis (y=0) and the lines x=a and x=b on each side, and by the curve y=f(x) on top.

    We can get approximations to the area of by dividing the interval into n equal parts (people don't always use equal parts, but we're just starting out!). Then we can look at "sample points" in each subinterval, compute f's value at those sample points, and multiply those heights by the widths of the subinterval to get areas of rectangles. Then (sigh!) finally take the sum of these rectangles, and that will be an approximation to the area we want to compute.

    The textbook explains in detail how to write the approximating sums. In particular, it identifies three sums corresponding to three choices of sample points: left and right endpoints of subintervals, and the midpoints of subintervals. Below are the formulas, which you should not memorize, and what are supposed to be pictures of a piece of the graph of y=f(x) over (relatively) tall and (relatively) narrow subintervals with the areas of the relevant boxes displayed. Since the interval has length b–a, the width of each subinterval is {b–a}/n. Do not memorize but do try to understand.

    Left-hand
    endpoints of
    subintervals
    (j=1nf(a+[{j–1}{b-a}/n]))({b–a}/n)
    Right-hand
    endpoints of
    subintervals
    (j=1nf(a+[{j}{b-a}/n]))({b–a}/n)
    Midpoints of
    subintervals
    (j=1nf(a+[{j–1}{b-a}/n+(1/2){b-a}/n]))({b–a}/n)

    QotD
    I don't remember! I think it was something like "Compute ∑j=13(j+{2/j})".


    Tuesday, November 10 (Lecture #20.5)
    This is the last half of this lecture, put into the 3rd diary file so that it doesn't confuse you when you are studying for the second exam.

    Antiderivatives
    A function F is an antiderivative of f if F´=f. I think the name itself is almost the definition. This turns out to be a very useful concept. It has accumulated other names. For example, F is also called an indefinite integral of f. The word "primitive" is also used, as in, "F is a primitive of f".

    Example
    I think we should begin with a very simple example. Look at f(x)=x–3, quite simple. Then F(x)=(1/2)x2–3x is an antiderivative of f. Why it this true? Well, just compute the derivative of F. The 2 in the exponent moves down, and cancels the 2 in the 1/2. And –3x differentiates to –3. So the claim is verified. Actually (and this is useful to note!), you can always verify a claim that F is an antiderivative of f by just differentiating.

    Some qualitative aspects of even this "simple" example are worth noticing. The line is negative to the left of x=3, so the parabola (which happens to have roots at 0 and 6) must be decreasing to the left of x=3. The line is positive to the right of x=3, and there the parabola is increasing. Certainly, when x=3, the parabola must have a minimum, because to the left its derivative is negative and to the right its derivative is positive. The transition in first derivative sign from – to + indicates a local minimum (First Derivative Test).

    More example and discussion
    Well, as several (more than several, actually) tried to tell me, there are other antidervatives. For example, (1/2)x2–3x+2 is also an antiderivative of f(x)=x–3. The graph now to the right I hope shows y=(1/2)x2–3x+2. It is the original parabola shoved up. I hope that you notice it has the same qualitative ({in|de}creasing) properties in the same intervals as the original parabola. And we could push the parabola down (I think y=(1/2)x2–3x–1.75 is displayed).
    Etc. What does this "Etc." mean here?

    +C
    If F(x) is an antiderivative of f(x), then F(x)+C is also an antiderivative of f(x) for any constant C. This is because "+C" differentiates to 0. On the other hand, if G(x) is any antiderivative of f(x), then the function F(x)–G(x) has derivative equal to f(x)–f(x), which is 0. The MVT asserts that such a function is a constant. So G(x) must be F(x)+C. So all antiderivatives of a function can be described in this way:
    (Any specific antiderivative)+C.

    Here is a first very brief list of antiderivatives. This information is obtained just be reversing some derivative formulas. Please notice that if you don't believe a given formula, you can always differentiate and check. Also, the "+C" is generally omitted from tables of this type because it takes room and "one" should know it is there.

    FunctionAn antiderivative
    xn,
    n not equal to –1
    (1/[n+1])x
    1/xln(x)
    sin(x)–cos(x)
    cos(x)sin(x)
    exex
    There will be many more entries!
    General comment You already should feel that taking derivatives of functions given by formulas is straightforward. This is true. The reverse is emphatically not true. It is very easy to write a formula whose antiderivative is difficult to compute (even with the help of computers!). It is even easy to write a formula which has no nice antiderivative described by formulas (although it is not easy to confirm that fact). A big chunk of Math 152 is spent on showing you certain ways of guessing (?) antiderivatives. But there are lots of tricks, and the subject is difficult.

    Notation
    There is special notation for antiderivatives. For reasons which are not clear now but which will be clear (I hope!) in about two weeks, and which come from hundreds of years ago (really, this notation is an abbreviation of a Latin word) we would write the phrase, "F(x) is an antiderivative of f(x)" as ∫f(x) dx=F(x)+C. So, for example, ∫3ex+x5dx=3ex+(1/6)x6+C.

    Example and vocabulary
    A function can certainly have many antiderivatives (hey, if it has one, it must have infinitely many!). So how can we separate the various antiderivatives? The idea and the vocabulary come from simple physical considerations, where the derivative is velocity and the function is position (and x represents time). If the velocity is prescribed, then we should be able to tell where the object is if the starting position is given.

    So this is the idea of an initial value problem. A differential equation is given for an "unknown" function, and then some value of the unknown function is given at some value of its argument (the initial condition). Let's look at a simple example.

    Initial Value Problem
        Differential equation y´=2x7+8x.
        Initial condition y(1)=3.
    Since y´=2x7+8x I know (consult the table above) that y must be (2/7)x8+4x2+C. I put in the "+C" so that all possibilities for y are covered. Now plug in x=1 and set this equal to the given value of y. We get: (2/7)+4+C=3, so C=3–4–(2/7)=–(9/7). The particular solution, the one specific solution for this initial value problem is therefore y=(2/7)x8+4x2–(9/7).

    The idea here is that the differential equation tells how a quantity changes or evolves (?) with respect to its independent variable, and that giving a starting value together with a description of the rate of change should be enough to deduce the value of the quantity at any other time. Of course, whether or not this is practical or gives any useful information is something we need to investigate. Here is a very direct version of this idea: if you tell me that my weight last January 1st was 180 lbs, and then you keep track of, say, my monthly gains and losses, you certainly should be able to tell my weight at the end of the year.

    Superman
    Suppose that Superman leaps straight up at 40 ft/sec from the top of a 20 ft tall hill. What is his maximum height, and when does he get highest? In this case, the presumed gravity of earth pulls down and we will say that g is –32 ft/sec2.

    It was my feeling that most students know how to do a problem like this using formulas, and I wanted to plod through it principally as a way of checking our vocabulary.

    Translation
    Let's measure height from ground level in feet, with + meaning above ground level. And we will measure time in seconds from when Superman jumped. What do we know? There are facts about three functions, and confusion is easy.

    1. Gravity causes changes in velocity. So gravity refers to acceleration, a(t), which is the derivative of velocity, v(t). Velocity is in turn the derivative of position, s(t). In this problem, a(t)=–32 (I will start omitting units here). Or we could write v´(t)=–32. Or we could write s´´(t)=–32. Wow.
    2. What do we know about velocity? Our main character "leaps straight up at 40 ft/sec" and this is at the initial time. So v(0)=40.
    3. And how about distance? Well, s(0)=20 since the leaping is done "from the top of a 20 ft tall hill".
    The translation is therefore, in as mathematical terms as possible, the following initial value problem:
        Differential equation s´´(t)=–32.
    This is a second order differential equation. The "second order" refers to the highest numbered derivative (second derivative in this case) which appears in the equation.)
        Initial conditions s´(0)=40 and s(0)=20.
    This problem has two initial conditions. You'll see that the simplest differential equations have exactly one solution when the number of initial conditions equals the order of the differential equation. The idea is that for each step of the order, we'll need to do one antidifferentiation. So here we'll need two antidifferentiations, and therefore two initial conditions. Also the physics works out correctly.

    Solution

    Comments on this problem and its solution
    This is not a difficult problem, and I am sure that lots of formulas memorized for physics can be used to get the answers. I want to be sure that you understand the language and the process, so that more interesting problems can be analyzed successfully.

    The graph to the right has time in seconds as the horizontal axis and has height in feet as the vertical axis. The units differ. I plotted the height functon for the time interval [0,3]. My Superman has somehow landed below ground level on his flight. Oh well. In fact, I think my Superman would hit the darn hill he leaped from (unless someone else has moved it during his flight) at time 2.5. I've never found this sort of problem too believable.

    Car acceleration
    So I need to buy a car. One car I am thinking about goes from 0 to 60 mph in 9.5 seconds. How many g's (g is acceleration of gravity) is this?

    How can we set this up as a differential equation problem? Again, right now, I am more interested in the process than the answer. Well, 60 mph is 88 feet per second. Assume we begin at t=0 seconds, with both position and velocity equal to 0: s(0)=0 and v(0)=0. We know that v(9.5)=88.

    Also assume acceleration is constant, so a(t)=K. Now v(t)=Kt+C, and (using v(0)=0) we get v(t)=Kt. Since v(9.5)=88, I see that K(9.5)=88, and K=88/9.5, approximately 9.263. Now g (in these units) is 32, so this car accelerates (for this task) at 9.263/32, approximately .289 g. And then I wondered what distance the car takes to get to 60 MPH, surely a relevant safety question. For this we need to know s(t). But s(t)=(1/2)Kt2+C, and again C–0 (since s(0)=0). So s(9.5)=(1/2)(9.263)(9.5)2, about 417.993 ft, or a bit less than a tenth of a mile (hey, more than a football field). This car's list price is $16,500.

    Now another car surely which should be bought by a faculty member will go from 0 to 60 in 3.4 seconds. Let's go through the numbers: K(3.4)=88, so this K is 25.882, and g is 25.882, about .81 g. Huh: about three times the acceleration. And the distance needed to get to 60 mph is s(3.4)=(1/2)(25.882)(3.4)2, about 150 feet, half the length of a football field. This list price of this reasonable vehicle is $123,600. Indeed.

    Which should I buy?


    Maintained by greenfie@math.rutgers.edu and last modified 11/17/2009.